hook-length formula: “Fibonaccized” Part I












16












$begingroup$


Consider the Young diagram of a partition $lambda = (lambda_1,ldots,lambda_k)$. For a square $(i,j) in lambda$, define the hook numbers $h_{(i,j)} = lambda_i + lambda_j' -i - j +1$ where $lambda'$ is the conjugate of $lambda$.



The hook-length formula shows, in particular, that if $lambdavdash n$ then
$$text{$n!prod_{square,in,lambda}frac1{h_{square}}$} qquad text{is an integer}.$$
Recall the Fibonacci numbers $F(0)=0, , F(1)=1$ with $F(n)=F(n-1)+F(n-2)$. Define $[0]!_F=1$ and $[n]!_F=F(1)cdot F(2)cdots F(n)$ for $ngeq1$.




QUESTION. Is it true that
$$text{$[n]!_Fprod_{square,in,lambda}frac1{F(h_{square})}$} qquad text{is an integer}?$$











share|cite|improve this question











$endgroup$








  • 5




    $begingroup$
    More generally, this interesting question can be asked of any strong divisibility sequence instead of the Fibonacci sequence. But let's perhaps not abuse the notation $Fleft(nright)!$ for something that's not the factorial of $Fleft(nright)$.
    $endgroup$
    – darij grinberg
    Apr 1 at 3:50








  • 1




    $begingroup$
    Maybe call it $F!(n)$ instead of $F(n)!$. How far has this been checked?
    $endgroup$
    – Noam D. Elkies
    Apr 1 at 3:51








  • 9




    $begingroup$
    Maybe this expression can be obtained by a clever substitution of the $q$-hook length formula?
    $endgroup$
    – Sam Hopkins
    Apr 1 at 4:30






  • 1




    $begingroup$
    @darijgrinberg what is a strong divisibility sequence? Product of any $k$ consecutive guys is divisibly by the product of first $k$ guys?
    $endgroup$
    – Fedor Petrov
    Apr 1 at 8:29






  • 5




    $begingroup$
    For searching purposes: the product of consecutive Fibonacci numbers is sometimes referred to as a fibonorial.
    $endgroup$
    – J. M. is not a mathematician
    Apr 1 at 10:52
















16












$begingroup$


Consider the Young diagram of a partition $lambda = (lambda_1,ldots,lambda_k)$. For a square $(i,j) in lambda$, define the hook numbers $h_{(i,j)} = lambda_i + lambda_j' -i - j +1$ where $lambda'$ is the conjugate of $lambda$.



The hook-length formula shows, in particular, that if $lambdavdash n$ then
$$text{$n!prod_{square,in,lambda}frac1{h_{square}}$} qquad text{is an integer}.$$
Recall the Fibonacci numbers $F(0)=0, , F(1)=1$ with $F(n)=F(n-1)+F(n-2)$. Define $[0]!_F=1$ and $[n]!_F=F(1)cdot F(2)cdots F(n)$ for $ngeq1$.




QUESTION. Is it true that
$$text{$[n]!_Fprod_{square,in,lambda}frac1{F(h_{square})}$} qquad text{is an integer}?$$











share|cite|improve this question











$endgroup$








  • 5




    $begingroup$
    More generally, this interesting question can be asked of any strong divisibility sequence instead of the Fibonacci sequence. But let's perhaps not abuse the notation $Fleft(nright)!$ for something that's not the factorial of $Fleft(nright)$.
    $endgroup$
    – darij grinberg
    Apr 1 at 3:50








  • 1




    $begingroup$
    Maybe call it $F!(n)$ instead of $F(n)!$. How far has this been checked?
    $endgroup$
    – Noam D. Elkies
    Apr 1 at 3:51








  • 9




    $begingroup$
    Maybe this expression can be obtained by a clever substitution of the $q$-hook length formula?
    $endgroup$
    – Sam Hopkins
    Apr 1 at 4:30






  • 1




    $begingroup$
    @darijgrinberg what is a strong divisibility sequence? Product of any $k$ consecutive guys is divisibly by the product of first $k$ guys?
    $endgroup$
    – Fedor Petrov
    Apr 1 at 8:29






  • 5




    $begingroup$
    For searching purposes: the product of consecutive Fibonacci numbers is sometimes referred to as a fibonorial.
    $endgroup$
    – J. M. is not a mathematician
    Apr 1 at 10:52














16












16








16


7



$begingroup$


Consider the Young diagram of a partition $lambda = (lambda_1,ldots,lambda_k)$. For a square $(i,j) in lambda$, define the hook numbers $h_{(i,j)} = lambda_i + lambda_j' -i - j +1$ where $lambda'$ is the conjugate of $lambda$.



The hook-length formula shows, in particular, that if $lambdavdash n$ then
$$text{$n!prod_{square,in,lambda}frac1{h_{square}}$} qquad text{is an integer}.$$
Recall the Fibonacci numbers $F(0)=0, , F(1)=1$ with $F(n)=F(n-1)+F(n-2)$. Define $[0]!_F=1$ and $[n]!_F=F(1)cdot F(2)cdots F(n)$ for $ngeq1$.




QUESTION. Is it true that
$$text{$[n]!_Fprod_{square,in,lambda}frac1{F(h_{square})}$} qquad text{is an integer}?$$











share|cite|improve this question











$endgroup$




Consider the Young diagram of a partition $lambda = (lambda_1,ldots,lambda_k)$. For a square $(i,j) in lambda$, define the hook numbers $h_{(i,j)} = lambda_i + lambda_j' -i - j +1$ where $lambda'$ is the conjugate of $lambda$.



The hook-length formula shows, in particular, that if $lambdavdash n$ then
$$text{$n!prod_{square,in,lambda}frac1{h_{square}}$} qquad text{is an integer}.$$
Recall the Fibonacci numbers $F(0)=0, , F(1)=1$ with $F(n)=F(n-1)+F(n-2)$. Define $[0]!_F=1$ and $[n]!_F=F(1)cdot F(2)cdots F(n)$ for $ngeq1$.




QUESTION. Is it true that
$$text{$[n]!_Fprod_{square,in,lambda}frac1{F(h_{square})}$} qquad text{is an integer}?$$








nt.number-theory co.combinatorics partitions algebraic-combinatorics






share|cite|improve this question















share|cite|improve this question













share|cite|improve this question




share|cite|improve this question








edited Apr 3 at 15:48







T. Amdeberhan

















asked Apr 1 at 3:42









T. AmdeberhanT. Amdeberhan

18.4k230132




18.4k230132








  • 5




    $begingroup$
    More generally, this interesting question can be asked of any strong divisibility sequence instead of the Fibonacci sequence. But let's perhaps not abuse the notation $Fleft(nright)!$ for something that's not the factorial of $Fleft(nright)$.
    $endgroup$
    – darij grinberg
    Apr 1 at 3:50








  • 1




    $begingroup$
    Maybe call it $F!(n)$ instead of $F(n)!$. How far has this been checked?
    $endgroup$
    – Noam D. Elkies
    Apr 1 at 3:51








  • 9




    $begingroup$
    Maybe this expression can be obtained by a clever substitution of the $q$-hook length formula?
    $endgroup$
    – Sam Hopkins
    Apr 1 at 4:30






  • 1




    $begingroup$
    @darijgrinberg what is a strong divisibility sequence? Product of any $k$ consecutive guys is divisibly by the product of first $k$ guys?
    $endgroup$
    – Fedor Petrov
    Apr 1 at 8:29






  • 5




    $begingroup$
    For searching purposes: the product of consecutive Fibonacci numbers is sometimes referred to as a fibonorial.
    $endgroup$
    – J. M. is not a mathematician
    Apr 1 at 10:52














  • 5




    $begingroup$
    More generally, this interesting question can be asked of any strong divisibility sequence instead of the Fibonacci sequence. But let's perhaps not abuse the notation $Fleft(nright)!$ for something that's not the factorial of $Fleft(nright)$.
    $endgroup$
    – darij grinberg
    Apr 1 at 3:50








  • 1




    $begingroup$
    Maybe call it $F!(n)$ instead of $F(n)!$. How far has this been checked?
    $endgroup$
    – Noam D. Elkies
    Apr 1 at 3:51








  • 9




    $begingroup$
    Maybe this expression can be obtained by a clever substitution of the $q$-hook length formula?
    $endgroup$
    – Sam Hopkins
    Apr 1 at 4:30






  • 1




    $begingroup$
    @darijgrinberg what is a strong divisibility sequence? Product of any $k$ consecutive guys is divisibly by the product of first $k$ guys?
    $endgroup$
    – Fedor Petrov
    Apr 1 at 8:29






  • 5




    $begingroup$
    For searching purposes: the product of consecutive Fibonacci numbers is sometimes referred to as a fibonorial.
    $endgroup$
    – J. M. is not a mathematician
    Apr 1 at 10:52








5




5




$begingroup$
More generally, this interesting question can be asked of any strong divisibility sequence instead of the Fibonacci sequence. But let's perhaps not abuse the notation $Fleft(nright)!$ for something that's not the factorial of $Fleft(nright)$.
$endgroup$
– darij grinberg
Apr 1 at 3:50






$begingroup$
More generally, this interesting question can be asked of any strong divisibility sequence instead of the Fibonacci sequence. But let's perhaps not abuse the notation $Fleft(nright)!$ for something that's not the factorial of $Fleft(nright)$.
$endgroup$
– darij grinberg
Apr 1 at 3:50






1




1




$begingroup$
Maybe call it $F!(n)$ instead of $F(n)!$. How far has this been checked?
$endgroup$
– Noam D. Elkies
Apr 1 at 3:51






$begingroup$
Maybe call it $F!(n)$ instead of $F(n)!$. How far has this been checked?
$endgroup$
– Noam D. Elkies
Apr 1 at 3:51






9




9




$begingroup$
Maybe this expression can be obtained by a clever substitution of the $q$-hook length formula?
$endgroup$
– Sam Hopkins
Apr 1 at 4:30




$begingroup$
Maybe this expression can be obtained by a clever substitution of the $q$-hook length formula?
$endgroup$
– Sam Hopkins
Apr 1 at 4:30




1




1




$begingroup$
@darijgrinberg what is a strong divisibility sequence? Product of any $k$ consecutive guys is divisibly by the product of first $k$ guys?
$endgroup$
– Fedor Petrov
Apr 1 at 8:29




$begingroup$
@darijgrinberg what is a strong divisibility sequence? Product of any $k$ consecutive guys is divisibly by the product of first $k$ guys?
$endgroup$
– Fedor Petrov
Apr 1 at 8:29




5




5




$begingroup$
For searching purposes: the product of consecutive Fibonacci numbers is sometimes referred to as a fibonorial.
$endgroup$
– J. M. is not a mathematician
Apr 1 at 10:52




$begingroup$
For searching purposes: the product of consecutive Fibonacci numbers is sometimes referred to as a fibonorial.
$endgroup$
– J. M. is not a mathematician
Apr 1 at 10:52










2 Answers
2






active

oldest

votes


















10












$begingroup$

Sam is correct of course about $q$-hook formula. Below is a short self-contained proof not relying on such advanced combinatorics.



Denote $h_1>ldots>h_k$ the set of hook lengths of the first column of diagram $lambda$. Then the multiset of hooks is $cup_{i=1}^k {1,2,ldots,h_i}setminus {h_i-h_j:i<j}$ and $n=sum_i h_i-frac{k(k-1)}2$.



Recall that $F(m)=P_m(alpha,beta)=prod_{d|m,d>1}Phi_d(alpha,beta)=prod_d (Phi_d(alpha,beta))^{eta_d(m)}$, where



$alpha,beta=(1pm sqrt{5})/2$;



$P_n(x,y)=x^{n-1}+x^{n-2}y+ldots+y^{n-1}$;



$Phi_d$ are homogeneous cyclotomic polynomials;



$eta_d(m)=chi_{mathbb{Z}}(m/d)$ (i.e., it equals to 1 if $d$ divides $m$, and to 0 otherwise).



Therefore it suffices to prove that for any fixed $d>1$ we have
$$
sum_{m=1}^n eta_d(m)+sum_{i<j}eta_d(h_i-h_j)-sum_{i=1}^ksum_{j=1}^{h_i}eta_d(j)geqslant 0.quad (ast)
$$

$(ast)$ rewrites as
$$
[n/d]+|i<j:h_iequiv h_j pmod d|-sum_{i=1}^k [h_i/d]geqslant 0.quad (bullet)
$$

LHS of $(bullet)$ does not change if we reduce all $h_i$'s modulo $d$ (and accordingly change $n=sum_i h_i-frac{k(k-1)}2$, of course), so we may suppose that $0leqslant h_ileqslant d-1$ for all $i$. For $a=0,1,dots, d-1$ denote $t_a=|i:h_i=a|$. Then $(bullet)$ rewrites as
$$
left[frac{-binom{sum_{i=0}^{d-1} t_i}2+sum_{i=0}^{d-1} it_i}dright]+
sum_{i=0}^{d-1} binom{t_i}2geqslant 0. quad (star)
$$



It remains to observe that LHS of $(star)$ equals to
$$
left[frac1dsum_{i<j}binom{t_i-t_j}2 right].
$$






share|cite|improve this answer











$endgroup$













  • $begingroup$
    Nice. So what does this integer count?
    $endgroup$
    – Brian Hopkins
    Apr 1 at 19:47










  • $begingroup$
    Yes, that was my plan to ask next.
    $endgroup$
    – T. Amdeberhan
    Apr 1 at 19:53












  • $begingroup$
    @Fedor: what are $alpha$ and $beta$? What's the connection between $P_m$ and $eta_d$, etc?
    $endgroup$
    – T. Amdeberhan
    Apr 2 at 18:36










  • $begingroup$
    @T.Amdeberhan sorry, forgot to copy the notations from my previous Fibonacci answer. Hope now it is clear.
    $endgroup$
    – Fedor Petrov
    Apr 2 at 18:58



















7












$begingroup$

This is a less elementary but maybe more conceptual proof, also giving some combinatorial meaning:
Use the formulas
$F(n) = frac{varphi^n -psi^n}{sqrt{5}}$, $varphi =frac{1+sqrt{5}}{2}, psi = frac{1-sqrt{5}}{2}$. Let $q=frac{psi}{varphi} = frac{sqrt{5}-3}{2}$, so that
$F(n) = frac{varphi^n}{sqrt{5}} (1-q^n)$



Then the Fibonacci hook-length formula becomes:



begin{align*}
f^{lambda}_F:= frac{[n]!_F}{prod_{uin lambda}F(h(u))} = frac{ varphi^{ binom{n+1}{2} } [n]!_q }{ varphi^{sum_{u in lambda} h(u)} prod_{u in lambda} (1-q^{h(u)})}
end{align*}

So we have an ordinary $q$-analogue of the hook-length formula. Note that
$$sum_{u in lambda} h(u) = sum_{i} binom{lambda_i}{2} + binom{lambda'_j}{2} + |lambda| = b(lambda) +b(lambda') +n$$
Using the $q-$analogue hook-length formula via major index (EC2, Chapter 21) we have



begin{align*}
f^lambda_F = varphi^{ binom{n}{2} -b(lambda)-b(lambda')} q^{-b(lambda)} sum_{Tin SYT(lambda)} q^{maj(T)} = (-q)^{frac12( -binom{n}{2} +b(lambda') -b(lambda))}sum_T q^{maj(T)}
end{align*}



Now, it is clear from the q-HLF formula that $q^{maj(T)}$ is a symmetric polynomial, with lowest degree term $b(lambda)$ and maximal degree $b(lambda) + binom{n+1}{2} - n -b(lambda) -b(lambda') =binom{n}{2} - b(lambda')$ so the median degree term is
$$M=frac12 left(b(lambda) +binom{n}{2} - b(lambda')right)$$
which cancels with the factor of $q$ in $f^{lambda}_F$, so the resulting polynomial is of the form
begin{align*}
f^{lambda}_F = (-1)^{M} sum_{T: maj(T) leq M } (q^{M-maj(T)} + q^{maj(T)-M}) \
= (-1)^{M} sum_{T} (-1)^{M-maj(T)}( varphi^{2(M-maj(T))} + psi^{2(M-maj(T)}) =
sum_T (-1)^{maj(T)} L(2(M-maj(T)))
end{align*}

where $L$ are the Lucas numbers.



**byproduct of collaborations with A. Morales and I. Pak.






share|cite|improve this answer











$endgroup$













  • $begingroup$
    Nice, thank you. This would be even more fitting to the 2nd part of this question mathoverflow.net/questions/327015/… Therefore, do you like to post it there as well?
    $endgroup$
    – T. Amdeberhan
    Apr 3 at 15:50










  • $begingroup$
    Thanks! I just pasted it there.
    $endgroup$
    – Greta Panova
    Apr 3 at 17:55












Your Answer








StackExchange.ready(function() {
var channelOptions = {
tags: "".split(" "),
id: "504"
};
initTagRenderer("".split(" "), "".split(" "), channelOptions);

StackExchange.using("externalEditor", function() {
// Have to fire editor after snippets, if snippets enabled
if (StackExchange.settings.snippets.snippetsEnabled) {
StackExchange.using("snippets", function() {
createEditor();
});
}
else {
createEditor();
}
});

function createEditor() {
StackExchange.prepareEditor({
heartbeatType: 'answer',
autoActivateHeartbeat: false,
convertImagesToLinks: true,
noModals: true,
showLowRepImageUploadWarning: true,
reputationToPostImages: 10,
bindNavPrevention: true,
postfix: "",
imageUploader: {
brandingHtml: "Powered by u003ca class="icon-imgur-white" href="https://imgur.com/"u003eu003c/au003e",
contentPolicyHtml: "User contributions licensed under u003ca href="https://creativecommons.org/licenses/by-sa/3.0/"u003ecc by-sa 3.0 with attribution requiredu003c/au003e u003ca href="https://stackoverflow.com/legal/content-policy"u003e(content policy)u003c/au003e",
allowUrls: true
},
noCode: true, onDemand: true,
discardSelector: ".discard-answer"
,immediatelyShowMarkdownHelp:true
});


}
});














draft saved

draft discarded


















StackExchange.ready(
function () {
StackExchange.openid.initPostLogin('.new-post-login', 'https%3a%2f%2fmathoverflow.net%2fquestions%2f326860%2fhook-length-formula-fibonaccized-part-i%23new-answer', 'question_page');
}
);

Post as a guest















Required, but never shown

























2 Answers
2






active

oldest

votes








2 Answers
2






active

oldest

votes









active

oldest

votes






active

oldest

votes









10












$begingroup$

Sam is correct of course about $q$-hook formula. Below is a short self-contained proof not relying on such advanced combinatorics.



Denote $h_1>ldots>h_k$ the set of hook lengths of the first column of diagram $lambda$. Then the multiset of hooks is $cup_{i=1}^k {1,2,ldots,h_i}setminus {h_i-h_j:i<j}$ and $n=sum_i h_i-frac{k(k-1)}2$.



Recall that $F(m)=P_m(alpha,beta)=prod_{d|m,d>1}Phi_d(alpha,beta)=prod_d (Phi_d(alpha,beta))^{eta_d(m)}$, where



$alpha,beta=(1pm sqrt{5})/2$;



$P_n(x,y)=x^{n-1}+x^{n-2}y+ldots+y^{n-1}$;



$Phi_d$ are homogeneous cyclotomic polynomials;



$eta_d(m)=chi_{mathbb{Z}}(m/d)$ (i.e., it equals to 1 if $d$ divides $m$, and to 0 otherwise).



Therefore it suffices to prove that for any fixed $d>1$ we have
$$
sum_{m=1}^n eta_d(m)+sum_{i<j}eta_d(h_i-h_j)-sum_{i=1}^ksum_{j=1}^{h_i}eta_d(j)geqslant 0.quad (ast)
$$

$(ast)$ rewrites as
$$
[n/d]+|i<j:h_iequiv h_j pmod d|-sum_{i=1}^k [h_i/d]geqslant 0.quad (bullet)
$$

LHS of $(bullet)$ does not change if we reduce all $h_i$'s modulo $d$ (and accordingly change $n=sum_i h_i-frac{k(k-1)}2$, of course), so we may suppose that $0leqslant h_ileqslant d-1$ for all $i$. For $a=0,1,dots, d-1$ denote $t_a=|i:h_i=a|$. Then $(bullet)$ rewrites as
$$
left[frac{-binom{sum_{i=0}^{d-1} t_i}2+sum_{i=0}^{d-1} it_i}dright]+
sum_{i=0}^{d-1} binom{t_i}2geqslant 0. quad (star)
$$



It remains to observe that LHS of $(star)$ equals to
$$
left[frac1dsum_{i<j}binom{t_i-t_j}2 right].
$$






share|cite|improve this answer











$endgroup$













  • $begingroup$
    Nice. So what does this integer count?
    $endgroup$
    – Brian Hopkins
    Apr 1 at 19:47










  • $begingroup$
    Yes, that was my plan to ask next.
    $endgroup$
    – T. Amdeberhan
    Apr 1 at 19:53












  • $begingroup$
    @Fedor: what are $alpha$ and $beta$? What's the connection between $P_m$ and $eta_d$, etc?
    $endgroup$
    – T. Amdeberhan
    Apr 2 at 18:36










  • $begingroup$
    @T.Amdeberhan sorry, forgot to copy the notations from my previous Fibonacci answer. Hope now it is clear.
    $endgroup$
    – Fedor Petrov
    Apr 2 at 18:58
















10












$begingroup$

Sam is correct of course about $q$-hook formula. Below is a short self-contained proof not relying on such advanced combinatorics.



Denote $h_1>ldots>h_k$ the set of hook lengths of the first column of diagram $lambda$. Then the multiset of hooks is $cup_{i=1}^k {1,2,ldots,h_i}setminus {h_i-h_j:i<j}$ and $n=sum_i h_i-frac{k(k-1)}2$.



Recall that $F(m)=P_m(alpha,beta)=prod_{d|m,d>1}Phi_d(alpha,beta)=prod_d (Phi_d(alpha,beta))^{eta_d(m)}$, where



$alpha,beta=(1pm sqrt{5})/2$;



$P_n(x,y)=x^{n-1}+x^{n-2}y+ldots+y^{n-1}$;



$Phi_d$ are homogeneous cyclotomic polynomials;



$eta_d(m)=chi_{mathbb{Z}}(m/d)$ (i.e., it equals to 1 if $d$ divides $m$, and to 0 otherwise).



Therefore it suffices to prove that for any fixed $d>1$ we have
$$
sum_{m=1}^n eta_d(m)+sum_{i<j}eta_d(h_i-h_j)-sum_{i=1}^ksum_{j=1}^{h_i}eta_d(j)geqslant 0.quad (ast)
$$

$(ast)$ rewrites as
$$
[n/d]+|i<j:h_iequiv h_j pmod d|-sum_{i=1}^k [h_i/d]geqslant 0.quad (bullet)
$$

LHS of $(bullet)$ does not change if we reduce all $h_i$'s modulo $d$ (and accordingly change $n=sum_i h_i-frac{k(k-1)}2$, of course), so we may suppose that $0leqslant h_ileqslant d-1$ for all $i$. For $a=0,1,dots, d-1$ denote $t_a=|i:h_i=a|$. Then $(bullet)$ rewrites as
$$
left[frac{-binom{sum_{i=0}^{d-1} t_i}2+sum_{i=0}^{d-1} it_i}dright]+
sum_{i=0}^{d-1} binom{t_i}2geqslant 0. quad (star)
$$



It remains to observe that LHS of $(star)$ equals to
$$
left[frac1dsum_{i<j}binom{t_i-t_j}2 right].
$$






share|cite|improve this answer











$endgroup$













  • $begingroup$
    Nice. So what does this integer count?
    $endgroup$
    – Brian Hopkins
    Apr 1 at 19:47










  • $begingroup$
    Yes, that was my plan to ask next.
    $endgroup$
    – T. Amdeberhan
    Apr 1 at 19:53












  • $begingroup$
    @Fedor: what are $alpha$ and $beta$? What's the connection between $P_m$ and $eta_d$, etc?
    $endgroup$
    – T. Amdeberhan
    Apr 2 at 18:36










  • $begingroup$
    @T.Amdeberhan sorry, forgot to copy the notations from my previous Fibonacci answer. Hope now it is clear.
    $endgroup$
    – Fedor Petrov
    Apr 2 at 18:58














10












10








10





$begingroup$

Sam is correct of course about $q$-hook formula. Below is a short self-contained proof not relying on such advanced combinatorics.



Denote $h_1>ldots>h_k$ the set of hook lengths of the first column of diagram $lambda$. Then the multiset of hooks is $cup_{i=1}^k {1,2,ldots,h_i}setminus {h_i-h_j:i<j}$ and $n=sum_i h_i-frac{k(k-1)}2$.



Recall that $F(m)=P_m(alpha,beta)=prod_{d|m,d>1}Phi_d(alpha,beta)=prod_d (Phi_d(alpha,beta))^{eta_d(m)}$, where



$alpha,beta=(1pm sqrt{5})/2$;



$P_n(x,y)=x^{n-1}+x^{n-2}y+ldots+y^{n-1}$;



$Phi_d$ are homogeneous cyclotomic polynomials;



$eta_d(m)=chi_{mathbb{Z}}(m/d)$ (i.e., it equals to 1 if $d$ divides $m$, and to 0 otherwise).



Therefore it suffices to prove that for any fixed $d>1$ we have
$$
sum_{m=1}^n eta_d(m)+sum_{i<j}eta_d(h_i-h_j)-sum_{i=1}^ksum_{j=1}^{h_i}eta_d(j)geqslant 0.quad (ast)
$$

$(ast)$ rewrites as
$$
[n/d]+|i<j:h_iequiv h_j pmod d|-sum_{i=1}^k [h_i/d]geqslant 0.quad (bullet)
$$

LHS of $(bullet)$ does not change if we reduce all $h_i$'s modulo $d$ (and accordingly change $n=sum_i h_i-frac{k(k-1)}2$, of course), so we may suppose that $0leqslant h_ileqslant d-1$ for all $i$. For $a=0,1,dots, d-1$ denote $t_a=|i:h_i=a|$. Then $(bullet)$ rewrites as
$$
left[frac{-binom{sum_{i=0}^{d-1} t_i}2+sum_{i=0}^{d-1} it_i}dright]+
sum_{i=0}^{d-1} binom{t_i}2geqslant 0. quad (star)
$$



It remains to observe that LHS of $(star)$ equals to
$$
left[frac1dsum_{i<j}binom{t_i-t_j}2 right].
$$






share|cite|improve this answer











$endgroup$



Sam is correct of course about $q$-hook formula. Below is a short self-contained proof not relying on such advanced combinatorics.



Denote $h_1>ldots>h_k$ the set of hook lengths of the first column of diagram $lambda$. Then the multiset of hooks is $cup_{i=1}^k {1,2,ldots,h_i}setminus {h_i-h_j:i<j}$ and $n=sum_i h_i-frac{k(k-1)}2$.



Recall that $F(m)=P_m(alpha,beta)=prod_{d|m,d>1}Phi_d(alpha,beta)=prod_d (Phi_d(alpha,beta))^{eta_d(m)}$, where



$alpha,beta=(1pm sqrt{5})/2$;



$P_n(x,y)=x^{n-1}+x^{n-2}y+ldots+y^{n-1}$;



$Phi_d$ are homogeneous cyclotomic polynomials;



$eta_d(m)=chi_{mathbb{Z}}(m/d)$ (i.e., it equals to 1 if $d$ divides $m$, and to 0 otherwise).



Therefore it suffices to prove that for any fixed $d>1$ we have
$$
sum_{m=1}^n eta_d(m)+sum_{i<j}eta_d(h_i-h_j)-sum_{i=1}^ksum_{j=1}^{h_i}eta_d(j)geqslant 0.quad (ast)
$$

$(ast)$ rewrites as
$$
[n/d]+|i<j:h_iequiv h_j pmod d|-sum_{i=1}^k [h_i/d]geqslant 0.quad (bullet)
$$

LHS of $(bullet)$ does not change if we reduce all $h_i$'s modulo $d$ (and accordingly change $n=sum_i h_i-frac{k(k-1)}2$, of course), so we may suppose that $0leqslant h_ileqslant d-1$ for all $i$. For $a=0,1,dots, d-1$ denote $t_a=|i:h_i=a|$. Then $(bullet)$ rewrites as
$$
left[frac{-binom{sum_{i=0}^{d-1} t_i}2+sum_{i=0}^{d-1} it_i}dright]+
sum_{i=0}^{d-1} binom{t_i}2geqslant 0. quad (star)
$$



It remains to observe that LHS of $(star)$ equals to
$$
left[frac1dsum_{i<j}binom{t_i-t_j}2 right].
$$







share|cite|improve this answer














share|cite|improve this answer



share|cite|improve this answer








edited Apr 2 at 18:57

























answered Apr 1 at 8:23









Fedor PetrovFedor Petrov

52.2k6122239




52.2k6122239












  • $begingroup$
    Nice. So what does this integer count?
    $endgroup$
    – Brian Hopkins
    Apr 1 at 19:47










  • $begingroup$
    Yes, that was my plan to ask next.
    $endgroup$
    – T. Amdeberhan
    Apr 1 at 19:53












  • $begingroup$
    @Fedor: what are $alpha$ and $beta$? What's the connection between $P_m$ and $eta_d$, etc?
    $endgroup$
    – T. Amdeberhan
    Apr 2 at 18:36










  • $begingroup$
    @T.Amdeberhan sorry, forgot to copy the notations from my previous Fibonacci answer. Hope now it is clear.
    $endgroup$
    – Fedor Petrov
    Apr 2 at 18:58


















  • $begingroup$
    Nice. So what does this integer count?
    $endgroup$
    – Brian Hopkins
    Apr 1 at 19:47










  • $begingroup$
    Yes, that was my plan to ask next.
    $endgroup$
    – T. Amdeberhan
    Apr 1 at 19:53












  • $begingroup$
    @Fedor: what are $alpha$ and $beta$? What's the connection between $P_m$ and $eta_d$, etc?
    $endgroup$
    – T. Amdeberhan
    Apr 2 at 18:36










  • $begingroup$
    @T.Amdeberhan sorry, forgot to copy the notations from my previous Fibonacci answer. Hope now it is clear.
    $endgroup$
    – Fedor Petrov
    Apr 2 at 18:58
















$begingroup$
Nice. So what does this integer count?
$endgroup$
– Brian Hopkins
Apr 1 at 19:47




$begingroup$
Nice. So what does this integer count?
$endgroup$
– Brian Hopkins
Apr 1 at 19:47












$begingroup$
Yes, that was my plan to ask next.
$endgroup$
– T. Amdeberhan
Apr 1 at 19:53






$begingroup$
Yes, that was my plan to ask next.
$endgroup$
– T. Amdeberhan
Apr 1 at 19:53














$begingroup$
@Fedor: what are $alpha$ and $beta$? What's the connection between $P_m$ and $eta_d$, etc?
$endgroup$
– T. Amdeberhan
Apr 2 at 18:36




$begingroup$
@Fedor: what are $alpha$ and $beta$? What's the connection between $P_m$ and $eta_d$, etc?
$endgroup$
– T. Amdeberhan
Apr 2 at 18:36












$begingroup$
@T.Amdeberhan sorry, forgot to copy the notations from my previous Fibonacci answer. Hope now it is clear.
$endgroup$
– Fedor Petrov
Apr 2 at 18:58




$begingroup$
@T.Amdeberhan sorry, forgot to copy the notations from my previous Fibonacci answer. Hope now it is clear.
$endgroup$
– Fedor Petrov
Apr 2 at 18:58











7












$begingroup$

This is a less elementary but maybe more conceptual proof, also giving some combinatorial meaning:
Use the formulas
$F(n) = frac{varphi^n -psi^n}{sqrt{5}}$, $varphi =frac{1+sqrt{5}}{2}, psi = frac{1-sqrt{5}}{2}$. Let $q=frac{psi}{varphi} = frac{sqrt{5}-3}{2}$, so that
$F(n) = frac{varphi^n}{sqrt{5}} (1-q^n)$



Then the Fibonacci hook-length formula becomes:



begin{align*}
f^{lambda}_F:= frac{[n]!_F}{prod_{uin lambda}F(h(u))} = frac{ varphi^{ binom{n+1}{2} } [n]!_q }{ varphi^{sum_{u in lambda} h(u)} prod_{u in lambda} (1-q^{h(u)})}
end{align*}

So we have an ordinary $q$-analogue of the hook-length formula. Note that
$$sum_{u in lambda} h(u) = sum_{i} binom{lambda_i}{2} + binom{lambda'_j}{2} + |lambda| = b(lambda) +b(lambda') +n$$
Using the $q-$analogue hook-length formula via major index (EC2, Chapter 21) we have



begin{align*}
f^lambda_F = varphi^{ binom{n}{2} -b(lambda)-b(lambda')} q^{-b(lambda)} sum_{Tin SYT(lambda)} q^{maj(T)} = (-q)^{frac12( -binom{n}{2} +b(lambda') -b(lambda))}sum_T q^{maj(T)}
end{align*}



Now, it is clear from the q-HLF formula that $q^{maj(T)}$ is a symmetric polynomial, with lowest degree term $b(lambda)$ and maximal degree $b(lambda) + binom{n+1}{2} - n -b(lambda) -b(lambda') =binom{n}{2} - b(lambda')$ so the median degree term is
$$M=frac12 left(b(lambda) +binom{n}{2} - b(lambda')right)$$
which cancels with the factor of $q$ in $f^{lambda}_F$, so the resulting polynomial is of the form
begin{align*}
f^{lambda}_F = (-1)^{M} sum_{T: maj(T) leq M } (q^{M-maj(T)} + q^{maj(T)-M}) \
= (-1)^{M} sum_{T} (-1)^{M-maj(T)}( varphi^{2(M-maj(T))} + psi^{2(M-maj(T)}) =
sum_T (-1)^{maj(T)} L(2(M-maj(T)))
end{align*}

where $L$ are the Lucas numbers.



**byproduct of collaborations with A. Morales and I. Pak.






share|cite|improve this answer











$endgroup$













  • $begingroup$
    Nice, thank you. This would be even more fitting to the 2nd part of this question mathoverflow.net/questions/327015/… Therefore, do you like to post it there as well?
    $endgroup$
    – T. Amdeberhan
    Apr 3 at 15:50










  • $begingroup$
    Thanks! I just pasted it there.
    $endgroup$
    – Greta Panova
    Apr 3 at 17:55
















7












$begingroup$

This is a less elementary but maybe more conceptual proof, also giving some combinatorial meaning:
Use the formulas
$F(n) = frac{varphi^n -psi^n}{sqrt{5}}$, $varphi =frac{1+sqrt{5}}{2}, psi = frac{1-sqrt{5}}{2}$. Let $q=frac{psi}{varphi} = frac{sqrt{5}-3}{2}$, so that
$F(n) = frac{varphi^n}{sqrt{5}} (1-q^n)$



Then the Fibonacci hook-length formula becomes:



begin{align*}
f^{lambda}_F:= frac{[n]!_F}{prod_{uin lambda}F(h(u))} = frac{ varphi^{ binom{n+1}{2} } [n]!_q }{ varphi^{sum_{u in lambda} h(u)} prod_{u in lambda} (1-q^{h(u)})}
end{align*}

So we have an ordinary $q$-analogue of the hook-length formula. Note that
$$sum_{u in lambda} h(u) = sum_{i} binom{lambda_i}{2} + binom{lambda'_j}{2} + |lambda| = b(lambda) +b(lambda') +n$$
Using the $q-$analogue hook-length formula via major index (EC2, Chapter 21) we have



begin{align*}
f^lambda_F = varphi^{ binom{n}{2} -b(lambda)-b(lambda')} q^{-b(lambda)} sum_{Tin SYT(lambda)} q^{maj(T)} = (-q)^{frac12( -binom{n}{2} +b(lambda') -b(lambda))}sum_T q^{maj(T)}
end{align*}



Now, it is clear from the q-HLF formula that $q^{maj(T)}$ is a symmetric polynomial, with lowest degree term $b(lambda)$ and maximal degree $b(lambda) + binom{n+1}{2} - n -b(lambda) -b(lambda') =binom{n}{2} - b(lambda')$ so the median degree term is
$$M=frac12 left(b(lambda) +binom{n}{2} - b(lambda')right)$$
which cancels with the factor of $q$ in $f^{lambda}_F$, so the resulting polynomial is of the form
begin{align*}
f^{lambda}_F = (-1)^{M} sum_{T: maj(T) leq M } (q^{M-maj(T)} + q^{maj(T)-M}) \
= (-1)^{M} sum_{T} (-1)^{M-maj(T)}( varphi^{2(M-maj(T))} + psi^{2(M-maj(T)}) =
sum_T (-1)^{maj(T)} L(2(M-maj(T)))
end{align*}

where $L$ are the Lucas numbers.



**byproduct of collaborations with A. Morales and I. Pak.






share|cite|improve this answer











$endgroup$













  • $begingroup$
    Nice, thank you. This would be even more fitting to the 2nd part of this question mathoverflow.net/questions/327015/… Therefore, do you like to post it there as well?
    $endgroup$
    – T. Amdeberhan
    Apr 3 at 15:50










  • $begingroup$
    Thanks! I just pasted it there.
    $endgroup$
    – Greta Panova
    Apr 3 at 17:55














7












7








7





$begingroup$

This is a less elementary but maybe more conceptual proof, also giving some combinatorial meaning:
Use the formulas
$F(n) = frac{varphi^n -psi^n}{sqrt{5}}$, $varphi =frac{1+sqrt{5}}{2}, psi = frac{1-sqrt{5}}{2}$. Let $q=frac{psi}{varphi} = frac{sqrt{5}-3}{2}$, so that
$F(n) = frac{varphi^n}{sqrt{5}} (1-q^n)$



Then the Fibonacci hook-length formula becomes:



begin{align*}
f^{lambda}_F:= frac{[n]!_F}{prod_{uin lambda}F(h(u))} = frac{ varphi^{ binom{n+1}{2} } [n]!_q }{ varphi^{sum_{u in lambda} h(u)} prod_{u in lambda} (1-q^{h(u)})}
end{align*}

So we have an ordinary $q$-analogue of the hook-length formula. Note that
$$sum_{u in lambda} h(u) = sum_{i} binom{lambda_i}{2} + binom{lambda'_j}{2} + |lambda| = b(lambda) +b(lambda') +n$$
Using the $q-$analogue hook-length formula via major index (EC2, Chapter 21) we have



begin{align*}
f^lambda_F = varphi^{ binom{n}{2} -b(lambda)-b(lambda')} q^{-b(lambda)} sum_{Tin SYT(lambda)} q^{maj(T)} = (-q)^{frac12( -binom{n}{2} +b(lambda') -b(lambda))}sum_T q^{maj(T)}
end{align*}



Now, it is clear from the q-HLF formula that $q^{maj(T)}$ is a symmetric polynomial, with lowest degree term $b(lambda)$ and maximal degree $b(lambda) + binom{n+1}{2} - n -b(lambda) -b(lambda') =binom{n}{2} - b(lambda')$ so the median degree term is
$$M=frac12 left(b(lambda) +binom{n}{2} - b(lambda')right)$$
which cancels with the factor of $q$ in $f^{lambda}_F$, so the resulting polynomial is of the form
begin{align*}
f^{lambda}_F = (-1)^{M} sum_{T: maj(T) leq M } (q^{M-maj(T)} + q^{maj(T)-M}) \
= (-1)^{M} sum_{T} (-1)^{M-maj(T)}( varphi^{2(M-maj(T))} + psi^{2(M-maj(T)}) =
sum_T (-1)^{maj(T)} L(2(M-maj(T)))
end{align*}

where $L$ are the Lucas numbers.



**byproduct of collaborations with A. Morales and I. Pak.






share|cite|improve this answer











$endgroup$



This is a less elementary but maybe more conceptual proof, also giving some combinatorial meaning:
Use the formulas
$F(n) = frac{varphi^n -psi^n}{sqrt{5}}$, $varphi =frac{1+sqrt{5}}{2}, psi = frac{1-sqrt{5}}{2}$. Let $q=frac{psi}{varphi} = frac{sqrt{5}-3}{2}$, so that
$F(n) = frac{varphi^n}{sqrt{5}} (1-q^n)$



Then the Fibonacci hook-length formula becomes:



begin{align*}
f^{lambda}_F:= frac{[n]!_F}{prod_{uin lambda}F(h(u))} = frac{ varphi^{ binom{n+1}{2} } [n]!_q }{ varphi^{sum_{u in lambda} h(u)} prod_{u in lambda} (1-q^{h(u)})}
end{align*}

So we have an ordinary $q$-analogue of the hook-length formula. Note that
$$sum_{u in lambda} h(u) = sum_{i} binom{lambda_i}{2} + binom{lambda'_j}{2} + |lambda| = b(lambda) +b(lambda') +n$$
Using the $q-$analogue hook-length formula via major index (EC2, Chapter 21) we have



begin{align*}
f^lambda_F = varphi^{ binom{n}{2} -b(lambda)-b(lambda')} q^{-b(lambda)} sum_{Tin SYT(lambda)} q^{maj(T)} = (-q)^{frac12( -binom{n}{2} +b(lambda') -b(lambda))}sum_T q^{maj(T)}
end{align*}



Now, it is clear from the q-HLF formula that $q^{maj(T)}$ is a symmetric polynomial, with lowest degree term $b(lambda)$ and maximal degree $b(lambda) + binom{n+1}{2} - n -b(lambda) -b(lambda') =binom{n}{2} - b(lambda')$ so the median degree term is
$$M=frac12 left(b(lambda) +binom{n}{2} - b(lambda')right)$$
which cancels with the factor of $q$ in $f^{lambda}_F$, so the resulting polynomial is of the form
begin{align*}
f^{lambda}_F = (-1)^{M} sum_{T: maj(T) leq M } (q^{M-maj(T)} + q^{maj(T)-M}) \
= (-1)^{M} sum_{T} (-1)^{M-maj(T)}( varphi^{2(M-maj(T))} + psi^{2(M-maj(T)}) =
sum_T (-1)^{maj(T)} L(2(M-maj(T)))
end{align*}

where $L$ are the Lucas numbers.



**byproduct of collaborations with A. Morales and I. Pak.







share|cite|improve this answer














share|cite|improve this answer



share|cite|improve this answer








edited Apr 4 at 1:57









darij grinberg

18.4k373189




18.4k373189










answered Apr 3 at 5:55









Greta PanovaGreta Panova

33124




33124












  • $begingroup$
    Nice, thank you. This would be even more fitting to the 2nd part of this question mathoverflow.net/questions/327015/… Therefore, do you like to post it there as well?
    $endgroup$
    – T. Amdeberhan
    Apr 3 at 15:50










  • $begingroup$
    Thanks! I just pasted it there.
    $endgroup$
    – Greta Panova
    Apr 3 at 17:55


















  • $begingroup$
    Nice, thank you. This would be even more fitting to the 2nd part of this question mathoverflow.net/questions/327015/… Therefore, do you like to post it there as well?
    $endgroup$
    – T. Amdeberhan
    Apr 3 at 15:50










  • $begingroup$
    Thanks! I just pasted it there.
    $endgroup$
    – Greta Panova
    Apr 3 at 17:55
















$begingroup$
Nice, thank you. This would be even more fitting to the 2nd part of this question mathoverflow.net/questions/327015/… Therefore, do you like to post it there as well?
$endgroup$
– T. Amdeberhan
Apr 3 at 15:50




$begingroup$
Nice, thank you. This would be even more fitting to the 2nd part of this question mathoverflow.net/questions/327015/… Therefore, do you like to post it there as well?
$endgroup$
– T. Amdeberhan
Apr 3 at 15:50












$begingroup$
Thanks! I just pasted it there.
$endgroup$
– Greta Panova
Apr 3 at 17:55




$begingroup$
Thanks! I just pasted it there.
$endgroup$
– Greta Panova
Apr 3 at 17:55


















draft saved

draft discarded




















































Thanks for contributing an answer to MathOverflow!


  • Please be sure to answer the question. Provide details and share your research!

But avoid



  • Asking for help, clarification, or responding to other answers.

  • Making statements based on opinion; back them up with references or personal experience.


Use MathJax to format equations. MathJax reference.


To learn more, see our tips on writing great answers.




draft saved


draft discarded














StackExchange.ready(
function () {
StackExchange.openid.initPostLogin('.new-post-login', 'https%3a%2f%2fmathoverflow.net%2fquestions%2f326860%2fhook-length-formula-fibonaccized-part-i%23new-answer', 'question_page');
}
);

Post as a guest















Required, but never shown





















































Required, but never shown














Required, but never shown












Required, but never shown







Required, but never shown

































Required, but never shown














Required, but never shown












Required, but never shown







Required, but never shown







Popular posts from this blog

If I really need a card on my start hand, how many mulligans make sense? [duplicate]

Alcedinidae

Can an atomic nucleus contain both particles and antiparticles? [duplicate]